From 51e9d43716398dcf0fd1185e14d02cad4de7be87 Mon Sep 17 00:00:00 2001 From: Adrian Kummerlaender Date: Tue, 14 Feb 2017 18:17:23 +0100 Subject: Add Makefile --- content/analysis.tex | 943 ++++++++++++++++++++++++++++++++++++++++++++ content/analysis_3.tex | 188 +++++++++ content/lineare_algebra.tex | 706 +++++++++++++++++++++++++++++++++ content/numerik_1.tex | 436 ++++++++++++++++++++ 4 files changed, 2273 insertions(+) create mode 100644 content/analysis.tex create mode 100644 content/analysis_3.tex create mode 100644 content/lineare_algebra.tex create mode 100644 content/numerik_1.tex (limited to 'content') diff --git a/content/analysis.tex b/content/analysis.tex new file mode 100644 index 0000000..f2e3edb --- /dev/null +++ b/content/analysis.tex @@ -0,0 +1,943 @@ +\section*{Folgen} + +$\displaystyle\lim_{n \to \infty} = a \Leftrightarrow \forall \epsilon > 0 \exists N_\epsilon \in \N \forall n \geq N_\epsilon : | a_n - a | \leq \epsilon$ + +\subsection*{Konvergenzsatz für monotone Folgen} + +Sei $(a_n)$ wachsend und nach oben beschränkt \\ $\Rightarrow \exists \lim_{n \to \infty} a_n = \sup_{n\geq 1} a_n := \sup\{a_n | n \in \N\}$ + +Analoges für fallende, nach unten beschr. Folgen. + +\subsection*{Bolzano-Weierstraß} + +Jede beschränkte Folge hat einen Häufungspunkt + +$\overline\lim_{x \to \infty} a_n$ Maximum der Häufungspunkte + +$\underline\lim_{x \to \infty} a_n$ Minimum der Häufungspunkte + +\subsection*{Cauchyfolgen} + +$\forall \epsilon > 0 \exists N_\epsilon \in \N \forall n, m \geq N_\epsilon : | a_n - a_m | \leq \epsilon$ + +Cauchyfolge $\Leftrightarrow$ konvergente Folge + +\subsection*{Uneigentliche Grenzwerte} + +Sei $\overline{\R} = \R \cup \{-\infty, +\infty\}$ + +$\forall K \in \N \exists N_K \in \N \forall n \geq N_K : x_n \geq K \Leftrightarrow \displaystyle\lim_{n \to \infty} x_n = \infty$ + +\subsection*{Beispiele und Hinweise} + +$$e^x = exp(x) = \lim_{n\to \infty} \Big(1 + \frac{x}{n}\Big)^n \text{ insb. } e = \lim_{n\to \infty} \Big(1 + \frac{1}{n}\Big)^n$$ + +Zur Bestimmung von Folgen Grenzwerten kann auch L'Hospital herangezogen werden. + +\section*{Reihen} + +Sei $(a_k)_{k\geq 0}$ Folge, dann ist $\sum_{k\geq 0} a_k$ Reihe. + +\subsection*{Konvergenzkriterien} + +$\sum_{k=0}^\infty a_k = \lim_{n\to \infty} \sum_{k=0}^n a_k = \lim_{n\to \infty} s_n$ + +$\sum_{k\geq 0} a_k$ konvergiert $\Rightarrow (a_k)$ ist Nullfolge. + +\subsubsection*{Leibnizkriterium} + +Es gelte $\forall k \in \N_0 : b_k \geq b_{k+1} \geq 0$ und $\displaystyle \lim_{k \to \infty} b_k = 0$ + +Dann konvergiert: $\sum_{k=0}^\infty (-1)^k b_k$ + +\subsubsection*{Majorantenkriterium} + +\begin{enumerate}[label=(\alph*)] + \item Wenn $0 \leq |a_k| \leq b_k \forall k \in \N_0$ und $\sum_k b_k$ konvergiert, dann konvergiert $\sum_k a_k$ absolut und es gilt $|\sum_{k=0}^\infty a_k| \leq \sum_{k=0}^\infty |a_k| \leq \sum_{k=0}^\infty b_k$. + \item Wenn $a_k \geq b_k \geq 0$ und $\sum_k b_k$ divergiert, dann divergiert $\sum_k a_k$ +\end{enumerate} + +\subsubsection*{Quotientenkriterium} + +Sei $(a_n)_{n\geq 0}$ Folge, $n_0 \in \N$ mit $\forall n \geq n_0 : a_n \neq 0$ + +$\overline\lim_{n \to \infty} |\frac{a_{n+1}}{a_n}| < 1 \implies \sum a_n$ konvergiert absolut + +$\underline\lim_{n \to \infty} |\frac{a_{n+1}}{a_n}| > 1 \implies \sum_{n\geq 0} a_n$ divergiert + +\subsubsection*{Wurzelkriterium} + +\vspace*{-4mm} +\begin{align*} + \overline\lim_{n \to \infty} \sqrt[n]{|a_n|} < 1 &\implies \textstyle\sum_{n\geq 1} a_n \text{ konvergiert} \\ + \overline\lim_{n \to \infty} \sqrt[n]{|a_n|} > 1 &\implies \textstyle\sum_{n\geq 1} a_n \text{ divergiert} \\ + (b_n) \text{ unbeschränkt } &\implies \textstyle\sum_{n\geq 0} a_n \text{ divergiert} +\end{align*} + +\subsubsection*{Integralkriterium} + +Für monoton fallende Koeffizientenfolge $f$ gilt: + +$\int_p^\infty f(n) dn$ integrierbar $\Leftrightarrow \sum_{n=p}^\infty f(n)$ konvergiert. + +\subsection*{Beispiele} + +\vspace*{-8mm} +\begin{align*} + \textstyle\sum_{n=0}^\infty \frac{x^n}{n!} &= e^x \text{ (insb. } \textstyle\sum_{k=0}^{\infty} \frac{1}{k!} = e) \\ + \textstyle\sum_{k=0}^{\infty} z^k &= \frac{1}{1-z} (z \in \mathbb{C} \land |z|<1) \\ + \textstyle\sum_{k=0}^{\infty} (\alpha a_k + \beta b_k) &= \alpha \textstyle\sum_{k=0}^{\infty} a_k + \beta \textstyle\sum_{k=0}^{\infty} b_k +\end{align*} +\vspace*{-4mm} + +Die allgemeine harmonische Reihe $\sum_{k=0}^\infty \frac{1}{k^\alpha}$ divergiert für $\alpha \leq 1$ und konvergiert für $\alpha > 1$. + +\subsection*{Cauchykriterium} + +Eine Reihe $\sum_k a_k$ konvergiert genau dann, wenn: + +$\forall \epsilon > 0 \exists N_\epsilon \in \N \forall n > m \geq N_\epsilon : | \sum_{k=m+1}^{n} a_k | \leq \epsilon$ + +\section*{Potenzreihen} + +Sei $z \in \mathbb{C}$, dann $\sum a_n z^n$ Potenzreihe mit Koeff. $a_n$. + +\begin{description}[leftmargin=!] + \item[Konvergenzradius] $p := \frac{1}{\overline\lim_{n \to \infty} \sqrt[n]{|a_n|}}$ +\end{description} + +\subsubsection*{Konvergenzsatz für Potenzreihen} + +Sei $p$ der Konvergenzradius von $\sum_n a_n z^n$, dann: + +\begin{description}[leftmargin=!,labelwidth=15mm] + \item[$p \in (0, \infty)$] $\sum_n a_n z^n \text{ konv. abs. für } |z| < p, \\ \text{divergiert für } |z| > p$ + \item[$p=0$] $\sum_n a_n z^n \text{ divergiert } \forall z \in \mathbb{C}\setminus \{0\}$ + \item[$p=\infty$] $\sum_n a_n z^n \text{ konvergiert abs. } \forall z \in \mathbb{C}$ +\end{description} + +\subsection*{Winkelfunktionen} + +\vspace*{-3mm} +\begin{align*} + sin(z) &= \textstyle\sum_{k=0}^\infty \frac{(-1)^k}{(2k+1)!} z^{2k+1} \\ + cos(z) &= \textstyle\sum_{k=0}^\infty \frac{(-1)^k}{(2k)!} z^{2k} +\end{align*} + +Auch: $tan(x) = \frac{sin(x)}{cos(x)}$ und $cot(x) = \frac{cos(x)}{sin(x)}$ + +\subsubsection*{Additionstheoreme und andere Hilfen} + +\vspace*{-3mm} +\begin{align*} + \sin(x\pm y) &= \sin(x) \cos(y) \pm \cos(x) \sin(y) \\ + \cos(x\pm y) &= \cos(x) \cos(y) \mp \sin(x) \sin(y) \\ + \tan(x\pm y) &= \textstyle\frac{\sin(x\pm y)}{\cos(x\pm y)} \\ + \cos(x) \cos(y) &= \textstyle\frac{1}{2} ( \cos(x-y) + \cos(x+y) ) \\ + \sin(x) \cos(y) &= \textstyle\frac{1}{2} ( \sin(x-y) + \sin(x+y) ) \\ + \sin^2(x) + \cos^2(x) &= 1 \\ + \cosh^2(x) - \sinh^2(x) &= 1 +\end{align*} + + +\section*{Stetige Funktionen} + +Eine Funktion $f : D \rightarrow \mathbb{C}$ ist stetig in $z_0 \in D$, wenn für jede Folge $(z_n) \subseteq D$ mit $z_n \rightarrow z_0$ auch $f(z_n) \rightarrow f(z_0)$ für $n \rightarrow \infty$ gilt. + +Wenn $z_0 \in D$ ein isolierter Punkt, dann ist $f$ in $z_0$ immer stetig. + +\subsection*{Abgeschlossenheit} + +$D \subseteq \mathbb{C}$ ist abgeschlossen, wenn $z_n \in D \text{ für } n \in \N \land \lim_{n \to \infty} z_n = z \implies z \in D$. + +\subsection*{Gleichmäßige Stetigkeit} + +Sei $D \subseteq \R, f : D \rightarrow \R$. $f$ ist glm. stetig gdw.: + +$\forall \epsilon > 0 \exists \delta > 0 \forall x, x_0 \in D : | x - x_0 | < \delta \\ \hspace*{4mm} \implies | f(x) - f(x_0) | < \epsilon$ + +\subsection*{Satz vom Maximum} + +Sei $D \subseteq \mathbb{C}$ abgeschlossen und beschränkt, $f: D \rightarrow \R$ stetig, dann nimmt $f$ auf $D$ ein Maximum und Minimum an, ist also insbesondere beschränkt. + +\subsection*{Zwischenwertsatz} + +Sei $f: [a, b] \rightarrow \R$ stetig, dann: + +$f([a, b]) = [\min_{[a, b]} f, \max_{[a, b]} f]$. + +Insbesondere: + +$\forall y \in [\min_{[a, b]} f, \max_{[a, b]} f] \exists x \in [a, b]: f(x)=y$ + +\subsection*{Nullstellensatz} + +Sei $f \in [a, b] \rightarrow \R$ stetig mit $f(a)f(b) \leq 0$, dann: + +$\exists x \in [a, b]: f(x) = 0$ + +\subsection*{Intervallsatz} + +Sei $I \subseteq \R$ ein Intervall und $f : I \rightarrow \R$ stetig. + +Dann ist $f(I)$ ein Intervall. + +\subsection*{Konvergenz} + +\subsubsection*{Punktweise Konvergenz} + +Fkt. Folge $(f_n)$ konv. punktweise gegen $f$, wenn: + +$\forall z \in D \forall \epsilon > 0 \exists N_{\epsilon, z} \in \N \forall n \geq N_{\epsilon, z} : | f_n(z) - f(z) | \leq \epsilon$ + +Dies ist äquivalent zu $\lim_{n\to \infty} f_n(x)=f(x)$. + +\subsubsection*{Gleichmäßige Konvergenz} + +Fkt. Folge $(f_n)$ konv. gleichmäßig gegen $f$, wenn: + +$\forall z \in D \exists N_\epsilon \in \N \forall n \geq N_\epsilon : \sup_{z \in D} | f_n(z) - f(z) | \leq \epsilon$ + +Dies ist äquivalent zu $\displaystyle\lim_{n\to \infty}(\sup_{z \in D}| f_n(z) - f(z) |) = 0$. + +\section*{Differentialrechnung} + +\subsection*{Differenzierbarkeit} + +Funktion $f : I \rightarrow \R$ ist in $x_0$ differenzierbar, wenn: + +$\lim_{x \to x_0} \frac{f(x) - f(x_0)}{x - x_0} =: f'(x_0) = \frac{df}{dx}(x_0)$ existiert. + +\subsection*{Regeln} + +\begin{description}[leftmargin=!,labelwidth=17mm] + \item[Ketten] $(g \circ f)'(x_0) =g'(f(x_0))f'(x_0)$ + \item[Produkt] $(\alpha f + \beta)'(x_0) = \alpha f'(x_0) + \beta g'(x_0)$ + \item[ ] $(fg)'(x_0) = f'(x_0)g(x_0) + f(x_0)g'(x_0)$ + \item[Quotienten] $(\frac{1}{g})'(x_0) = - \frac{g'(x_0)}{g(x_0)^2}$ + \item[ ] $(\frac{f}{g})'(x_0) = \frac{f'(x_0)g(x_0) - f(x_0)g'(x_0)}{g(x_0)^2}$ +\end{description} + +\subsection*{Nützliche Ableitungen $\frac{d}{dx}$} + +\vspace*{-4mm} +\begin{multicols}{2} +\begin{description}[leftmargin=!,labelwidth=14mm] + \item[$\sqrt{x}$] $\frac{1}{2\sqrt{x}}$ + \item[$\sin(x)$] $\cos(x)$ + \item[$\cos(x)$] $-\sin(x)$ + \item[$\tan(x)$] $\frac{1}{\cos^2(x)}$ + \item[$\arcsin(x)$] $\frac{1}{\sqrt{1-x^2}}$ + \item[$\arccos(x)$] $\frac{-1}{\sqrt{1-x^2}}$ + \item[$\arctan(x)$] $\frac{1}{1+x^2}$ + \item[$\cosh(x)$] $\sinh(x)$ + \item[$\sinh(x)$] $\cosh(x)$ + \item[$a^x$] $\ln(a) a^x$ + \item[$e^{kx}$] $ke^{kx}$ + \item[$x^x$] $x^x(1+\ln(x))$ + \item[$\ln(x)$] $\frac{1}{x}$ + \item[$\log_a(x)$] $\frac{1}{\ln(a) x}$ + \item[$\ln(f(x))$] $\frac{f'(x)}{f(x)}$ + \item[$f(x)^{g(x)}$] $(e^{g(x)\ln(f(x))})'$ +\end{description} +\end{multicols} +\vspace*{-4mm} + +\subsection*{Mittelwertsatz} + +Sei $a < b$ in $\R$ und $f, g \in C^1((a, b),\R)$, dann: + +$\exists \xi \in (a, b) : ( f(b) - f(a) ) g'(\xi) = f'(\xi)(g(b) - g(a))$ + +Für die Funktion $g(x)=x$ folgt der Mittelwertsatz: + +$\exists \xi \in (a, b) : f'(\xi) = \frac{f(b) - f(a)}{b - a}$ + +Für $f(a)=f(b)$ existiert $f'(\xi)=0$ (Satz von Rolle). + +\subsection*{Konvexität} + +Verbindungsline zweier Punke liegt über Bild. + +d.h. $f'$ wächst in $I$, also $\forall x \in I : f''(x) \geq 0$ + +\subsection*{Konkavität} + +Verbindungsline zweier Punke liegt unter Bild. + +d.h. $f'$ fällt in $I$, also $\forall x \in I : f''(x) \leq 0$ + +\subsection*{Youngsche Ungleichung} + +Seien $x, y > 0$, $p \in (1, \infty)$ und $p' := \frac{p}{p-1}$, also $\frac{1}{p}+\frac{1}{p'}=1$. Dann gilt: $xy \leq \frac{1}{p}x^p + \frac{1}{p'}y^{p'}$ + +\subsection*{L'Hospital} + +Sei $-\infty \leq a < b \leq +\infty$, $f, g : (a, b) \rightarrow \R$ differenzierbar mit $\forall x \in (a, b) : g'(x) \neq 0$ und es gelte eines: + +\begin{enumerate}[label=(\alph*)] + \item $\exists \lim_{x \to b^-} f(x) = \lim_{x \to b^-} g(x) = 0$ + \item $\exists \lim_{x \to b^-} g(x) = \pm \infty$ +\end{enumerate} + +Ferner existiere $l \in \overline{\R}$ mit $\lim_{x \to b^-} \frac{f'(x)}{g'(x)} = l$. + +Dann existiert $\lim_{x \to b^-} \frac{f(x)}{g(x)} = l$. + +\subsection*{Umkehrregel} + +Sei $f : I \subseteq \R \to \R$ strikt monoton, stetig und differenzierbar in $x_0 \in I$ mit $f'(x_0) \neq 0$. Dann ist $f^{-1} : f(I) \to \R$ in $y_0 = f(x_0)$ differenzierbar mit: + +$(f^{-1})'(y_0) = \frac{1}{f'(f^{-1}(y_0))} = \frac{1}{f'(x_0)}$ + +\section*{Taylorpolynome} + +$n$-tes Taylorpolynom von $f$ in $x_0$: + +$T_{n,x_0} f(x) = \sum_{j=0}^n \frac{f^{(j)}(x_0)}{j!} (x-x_0)^j$ + +\subsection*{Taylorreihe} + +Taylorreihe von $f$ in $x_0$: + +$T_{x_0} f(x) := \sum_{n\geq0} \frac{f^{(n)}(x_0)}{n!} (x-x_0)^n$ + +\subsection*{Lagrangesche Restgliedformel} + +$R_{n-1, x_0} f(x) = \frac{1}{n!} f^{(n)} (x_0 + \theta(x-x_0))(x-x_0)^n$ + +\subsection*{Integralrestglied} + +Sei $f \in C^{n+1}((a, b)), n \in \N_0, x_0 \in (a, b)$, dann gilt: + +$f(x) - T_{n,x_0} f(x) = \frac{1}{n!} \int_{x_0}^x (x-t)^n f^{(n+1)}(t) dt$ + +\section*{Integralrechnung} + +\subsection*{Stückweise Stetigkeit} + +$f : [a, b] \rightarrow \R$ ist stückweise stetig, wenn sich $[a, b]$ in endlich viele disjunkte Teilintervalle so aufteilen lässt, dass $f$ eingeschränkt auf diese stetig ist. + +$PC([a, b])$ ist Menge stckw. stg. Fkt. über $[a, b]$. + +\subsection*{Hauptsatz} + +Sei $f \in C([a, b])$, dann: $\int_a^b f(t) dt = F(b) - F(a) =: F|_a^b$ + +Sei $g \in C^1([a, b])$, dann: $\int_a^b g'(t) dt = g(b) - g(a)$ + +\subsection*{Partielle Integration} + +Sei $f, g \in C^1([a, b])$, dann: + +$\int_a^b f'(x)g(x) dx = (fg)|_a^b - \int_a^b f(x)g'(x) dx$ + +\subsection*{Substitutionsregel} + +Sei $\phi \in C^1([a, b]), J = \phi([a,b]), f \in C(J)$, dann gilt: + +$\int_a^b f(\phi(x))\phi'(x) dx = \int_{\phi(a)}^{\phi(b)} f(y) dy$ + +\subsection*{Uneigentliche Riemann-Integrale} + +Sei $-\infty < a < b \leq +\infty, f : [a, b) \rightarrow \R$ so, dass $\forall \beta \in (a, b): f|_{[a, \beta]} \in PC([a, \beta])$. + +Falls $\lim_{\beta \to b^-} \int_a^\beta f(x) dx =: \int_a^b f(x) dx$ in $\R$ existiert, heißt $f$ uneigentlich Riemann-integrierbar. + +\subsubsection*{Majoranten- / Minorantenkriterium} + +Sei $-\infty < a < b \leq +\infty$ und $f, g : [a, b) \to \R$ mit $\forall c \in (a, b) : f, g \in PC([a, c])$, dann: + +\begin{enumerate}[label=(\alph*)] + \item $\forall x \in [a, b) : |f(x)| \leq g(x) \land g$ uneigentlich integrierbar $\Rightarrow$ $f, |f|$ uneigentlich integrierbar. + \item $\forall x \in [a, b): f(x) \geq g(x) \geq 0 \land g$ nicht uneigentlich integrierbar $\Rightarrow f$ nicht uneigentlich integrierbar. +\end{enumerate} + +\section*{Eine Auswahl von Bildern} + +\begin{tikzpicture} +\begin{axis}[ + axis lines*=middle, + domain=-2.5:4.5, + samples=400, + ymin=-3, + ymax=10, + height=7cm, + width=8.5cm +] + \addplot[mark=none]{ln(x)} node [pos=1, above left] {$\ln(x)$}; + \addplot[mark=none]{e^x} node [pos=0.1, above left] {$e^x$}; + \addplot[mark=none]{x} node [pos=1, above left] {$x$}; + \addplot[mark=none]{sinh(x)} node [pos=0.3, below right] {$\sinh(x)$}; + \addplot[mark=none]{cosh(x)} node [pos=0, below right] {$\cosh(x)$}; + \addplot[mark=none]{sin(deg(x))}; +\end{axis} +\end{tikzpicture} + +\section*{Normierte und metrische Räume} + +\subsection*{Normen} + +\begin{description}[leftmargin=!,labelwidth=35mm] + \item[Definitheit] $||x|| = 0 \Leftrightarrow x = 0$ + \item[Homogenität] $||\alpha x|| = |\alpha| ||x||$ + \item[Dreiecksungleichung] $||x+y|| \leq ||x|| + ||y||$ +\end{description} + +\subsubsection*{Konvergenz} + +Folge $(x_n) \subseteq X$ konvergiert gegen $x \in X$, wenn: + +$\forall \epsilon > 0 \exists N_\epsilon \in \N \forall n \geq N_\epsilon : ||x_n - x|| \leq \epsilon$ + +\subsubsection*{$p$-Norm} + +$$|x|_p := \begin{cases} + (\sum_{k=1}^m |x_k|^p )^{\frac{1}{p}} & 1 \leq p < \infty \\ + \max_{1\leq k \leq m} |x_k| & p = \infty +\end{cases}$$ + +\subsubsection*{Hölder-Ungleichung} + +Sei $p \in |1, \infty]$ mit $\frac{1}{p}+\frac{1}{p'}=1$ sowie $x, y \in \K^m$: + +$|\sum_{k=1}^m x_k y_k| \leq \sum_{k=1}^m |x_k y_k| \leq |x|_p |x|_{p'}$ + +\subsubsection*{Minkowski-Ungleichung} + +Sei $x, y \in \K^m$: $|x+y|_p \leq |x|_p + |y|_p$ + +\subsubsection*{Cauchy-Schwarz-Ungleichung} + +Sei $(x|y) = \sum_{k=1}^m x_k \overline y_k$ Skalarprodukt, dann: + +$|(x|y)| \leq \sum_{k=1}^m |x_k y_k| \leq |x|_2 |y|_2$ + +\subsubsection*{Kugeln} + +\begin{description}[leftmargin=!,labelwidth=22mm] + \item[offen] $B(x,r) = \{y \in X | ||x-y|| < r\}$ + \item[abgeschlossen] $\overline B(x,r) = \{y \in X | ||x-y|| \leq r\}$ + \item[Sphäre] $S(x,r) = \{y \in X | ||x-y|| = r\}$ +\end{description} + +\subsubsection*{Äquivalenz} + +$||\cdot|| \sim |||\cdot||| \Leftrightarrow \exists C, c > 0 \forall x \in X \land r > 0 : \\ \hspace*{5mm} \overline B_{|||\cdot|||}(x, \frac{r}{C}) \subseteq B_{||\cdot||}(x, \frac{r}{C}) \subseteq B_{|||\cdot|||}(x, \frac{r}{c})$ + +\subsubsection*{Konvergenz bezüglich $p$-Normen} + +Sei $1 \leq p < q \leq \infty$, $x \in \K^m$, dann gilt: + +$|x|_p \leq m^{\frac{1}{p} - \frac{1}{q}} |x|_q$ und $|x|_q \leq |x|_p$ + +\subsubsection*{Cauchyfolgen bzgl. Normen} + +Sei $(X, ||\cdot||)$ normierter Vektorraum. $(x_n) \subseteq X$ ist Cauchyfolge, wenn gilt: + +$\forall \epsilon > 0 \exists N_\epsilon \in \N \forall n, m \geq N_\epsilon : ||x_n - x_m|| \leq \epsilon$ + +\subsection*{Normen stetiger Funktionen} + +$C([a, b])$ ist Vektorraum mit $dim(C([a, b])) = \infty$. + +Für $f \in C([a, b])$ gilt: + +\begin{description}[leftmargin=!,labelwidth=15mm] + \item[Supnorm] $||f||_\infty = \displaystyle\sup_{a\leq t \leq b} |f(t)| = \displaystyle\max_{a\leq t\leq b} |f(t)|$ + \item[1-Norm] $||f||_1 = \int_a^b |f(t)| dt$ +\end{description} + +\subsubsection*{Banachraum} + +Wenn jede Cauchyfolge in $(X, ||\cdot||)$ einen Grenzwert besitzt, dann heißt $||\cdot||$ vollständig und $(X, ||\cdot||)$ ist Banachraum. + +\subsubsection*{Hilbertraum} + +Die Norm des Banachraums $(\K^m, |\cdot|_2)$ ist durch Skalarprodukt gegeben. Ein solcher Banachraum heißt Hilbertraum. + +Ein $\R$-VRaum mit SKP ist ein euklidischer Raum. + +\subsubsection*{Äquivalenz der Normen} + +Sei $V$ endlichdim. VRaum, dann sind alle Normen auf $V$ äquivalent. Insb. ist $V$ ein Banachraum. + +\subsection*{Metriken} + +\begin{description}[leftmargin=!,labelwidth=35mm] + \item[Definitheit] $d(x, y) = 0 \Leftrightarrow x = y$ + \item[Symmetrie] $d(x, y) = d(y, x)$ + \item[Dreiecksungleichung] $d(x, z) \leq d(x, y) + d(y, z)$ +\end{description} + +\subsubsection*{Konvergenz bzgl. Metriken} + +$(x_n) \subseteq X$ konvergiert in $(M, d)$ gegen $x \in X$, wenn: + +$\forall \epsilon > 0 \exists N_\epsilon \in \N \forall n \geq N_\epsilon : d(x_n, x) \leq \epsilon$ + +\subsubsection*{Cauchyfolgen bzgl. Metriken} + +Eine Folge $(x_n) \subseteq M$ heißt Cauchfolge, wenn: + +$\forall \epsilon > 0 \exists N_\epsilon \in \N \forall n, m \geq N_\epsilon : d(x_n, x_m) \leq \epsilon$ + +Ein metrischer Raum ist vollständig, wenn jede Cauchfolge in $(M, d)$ konvergiert. + +\section*{Topologische Grundbegriffe} + +Sei $M$ ein metrischer Raum, dann: + +\begin{enumerate}[label=(\alph*)] + \item $O \subseteq M$ ist offen in $M$, wenn $\forall x \in O \exists r = r(x) > 0 : B(x, r) \subseteq O$ + \item $U \subseteq M$ ist Umgebung in $M$ von $x \in M$, wenn $\exists r > 0 : B(x, r) \subseteq U$ + \item $A \subseteq M$ ist abgeschlossen in $M$, wenn $M\setminus A$ in $M$ offen ist +\end{enumerate} + +\subsection*{Charakterisierung} + +Sei $M$ metrischer Raum und $A, O \subseteq M$, dann gilt: + +\begin{enumerate}[label=(\alph*)] + \item $A$ ist abgeschlossen in $M$ gdw. $(x_n \in A \land \lim_{n \to \infty} x_n = x \in M ) \Rightarrow x \in A$ + \item $O$ ist offen in $M$ gdw. $\nexists x \in O : \exists y_n \in M \setminus O : \lim_{n \to \infty} y_n = x$ +\end{enumerate} + +$M$ und $\emptyset$ sind offen und abgeschlossen in $M$. + +Die Vereinigung beliebig vieler und der Durschnitt endlich vieler offener Teilmengen von $M$ sind wieder offen in $M$. + +Der Durschnitt beliebig vieler und die Vereinigung endlich vieler abgeschlossener Teilmengen von $M$ sind wieder abgeschlossen in $M$. + +\subsection*{Inneres, Abschluss und Rand} + +Sei $M$ metrischer Raum und $\emptyset \neq N \subseteq M$, dann ist: + +\begin{description}[leftmargin=!,labelwidth=15mm] + \item[Innere] $N^\circ = \bigcup \{ O \subseteq N | O \text{ offen in } M \}$ + \item[Abschluss] $\overline N = \bigcap\{A \subseteq M | A \text{ abg. in } M, N \subseteq A\}$ + \item[Rand] $\partial N = \overline N \setminus N^\circ = \overline N \bigcap (M \setminus N^\circ)$ +\end{description} + +$N^\circ$ ist die größte in $M$ offene Teilmenge von $N$. + +Die Menge $N$ ist in $M$ offen gdw. $N = N^\circ$. + +\vspace*{-6mm} +\begin{align*} + N^\circ &= \{x \in M | \exists r > 0 : B(x, r) \subseteq N\} \\ + &= \{x \in M | \nexists (x_n) \in M \setminus N : \displaystyle\lim_{n \to \infty} x_n = x\} +\end{align*} +\vspace*{-6mm} + +$\overline N$ ist die kleinste in $M$ abg. Obermenge von $N$. + +Die Menge $N$ ist in $M$ abgeschlossen gdw. $N = \overline N$ + +$\overline N = N^\circ \cup \partial N = \{x \in M | \exists x_n \in N : \lim_{n \to \infty} x_n = x \}$ + +$\partial N$ ist abgeschlossen in $M$. + +$\partial N = \{ x \in M \\ \hspace*{8.5mm} | \exists x_n \in N, y_n \in M \setminus N : \displaystyle\lim_{n \to \infty} x_n = \displaystyle\lim_{n \to \infty} y_n = x \}$ + +\subsection*{Dichtheit} + +$N$ dicht in $M$ $\Leftrightarrow \forall x \in M \exists (x_n) \in N : \displaystyle\lim_{n \to \infty} x_n = x$ + +\subsection*{Stetigkeit} + +Seien $M, N$ metrische Räume, $f : M \rightarrow N$. + +\subsubsection*{Gleichmäßige Stetigkeit} + +$\forall \epsilon > 0 \exists \delta_\epsilon > 0 \forall x, y \in M : d_M(x, y) \leq \delta_\epsilon \Rightarrow d_N(f(x), f(y)) \leq \epsilon$ + +\subsubsection*{Lipschitz Stetigkeit} + +$f$ ist Lipschitz stetig mit Konstante $L > 0$, wenn: + +$\forall x, y \in M : d_N(f(x), f(y)) \leq L d_M(x, y)$ + +Lipschitz stetige Fkt. sind insb. auch glm. stetig. + +$L(V, W) = \{T : V \rightarrow W | T \text{ linear} \}$ sind lineare Abb. zwischen $\K$-VR $V$ und $W$, genannt Operatoren. + +\subsubsection*{Operatornorm} + +Seien $V$, $W$, $Z$ endlichdim. norm. VRäume, $T \in L(V, W)$ und $S \in L(W, Z)$. Dann gelten: + +\begin{enumerate}[label=(\alph*)] + \item $T$ ist Lipschitz stetig mit Konstante \\ $||T|| = ||T||_{L(V, W)} := \sup_{x\in V \setminus \{O\}} \frac{||Tx||}{||x||}$ + \item $\forall x \in V : ||Tx|| \leq ||T|| ||x||$ + \item $||\cdot||_{L(V,W)}$ heißt Operatornorm + \item $||ST|| \leq ||S|| ||T||$ + \item $L(V, W)$ ist Banachrm. bzgl. Operatornorm +\end{enumerate} + +\subsubsection*{Banachscher Fixpunktsatz} + +Sei $M$ metr. Raum und $f : M \rightarrow M$ strikte Kontraktion, d.h. $\exists q \in [0, 1) \forall x,y \in M : d(f(x), f(y)) \leq q d(x,y)$. Dann $\exists! x_* \in M : f(x_*) = x_*$ + +\subsection*{Kompaktheit} + +Eine Teilmenge eines endlichdim. norm. Vektorraum ist nach Bolzano-Weierstraß kompakt gdw. sie beschränkt und abgeschlossen ist. + +\subsubsection*{Homöomorphismus} + +Seien $K$, $N$ metrische Räume, $K$ kompakt, $A \subseteq K$ abgeschlossen, $f: K \rightarrow N$ stetig. Dann gilt: + +$f(A)$ ist in $N$ kompakt. + +$f$ injektiv $\Rightarrow f^{-1} : f(K) \rightarrow K$ stetig + +$f$ bijektiv $\Rightarrow f$ ist homömorph, d.h. bijektiv, stetig und hat stetige Umkehrabbildung. + +\subsubsection*{Satz vom Maximum} + +Sei $K$ komp. metr. Raum, $f: K \rightarrow \R$ stetig, dann: + +$\exists x_{\pm} \in K : f(x_+) = \displaystyle\max_{x \in K} f(x) \land f(x_-) = \displaystyle\min_{x \in K} f(x)$ + +\subsubsection*{Wegzusammenhang} + +Metr. Raum $M$ ist wegzusammenhängend gdw.: + +$\forall x, y \in M \exists w \in C([0,1], M) : w(0) = x \land w(1) = y$ + +\subsubsection*{Zwischenwertsatz} + +Seien $M$, $N$ metr. Räume, $M$ wegzusammenhängend und $f \in C(M, N)$. Dann ist $f(M)$ wegzusammenhängend. Für $N = \R$ ist $f(M)$ ein Intervall. + +\section*{Differentialrechnung in VRäumen} + +\subsection*{Kurventangente} + +Sei $J$ ein Intervall. Ein Weg ist stetige Abbildung $f = (f_1 \hdots f_m)^T : J \rightarrow \R^m$. Das Bild $\Gamma = f(J)$ heißt Kurve. $f$ ist Parametrisierung von $\Gamma$. + +Die Ableitung eines $C^1$-Wegs $f$ in $t_0 \in J$ ist def.: + +$f'(t_0) = \displaystyle\lim_{h \to 0} \frac{1}{h} (f(t_0 + h) - f(t_0)) = \begin{pmatrix} f_1'(t_0) \\ \vdots \\ f_m'(t_0)\end{pmatrix} \in \R^m$ + +Für $f'(t_0) \neq 0$ ist Tangente $T(\R)$ an $f(t_0)$ durch $T(t) = f(t_0) + (t - t_0)f'(t_0)$ mit $t \in \R$ gegeben. + +\subsection*{Partielle Ableitungen} + +$\partial_j f(x) = \partial_{x_j} f(x) = \frac{\partial f}{\partial x_j}(x) := \displaystyle\lim_{t \to 0} \frac{1}{t}(f(x+te_j)-f(x))$ + +\subsection*{Die Ableitung} + +Seien $V$, $W$ endlichdim. norm. VRäume, $D \subseteq V$ offen, $f: D \rightarrow W$, $x_0 \in D$, $r > 0$ mit $B_V(x_0, r) \subseteq D$ und betrachte $h \in V$ mit $0 < ||h||_V < r$. $f$ heißt differenzierbar in $x_0$, wenn $\exists A \in L(V, W)$ so, dass: + +$\displaystyle\lim_{||h||_V \to 0} \frac{1}{||h||_V}||f(x_0 + h) - f(x_0) - Ah||_W = 0$ + +$f'(x_0) := A$ ist Ableitung von $f$ bei $x_0$. Wenn $\forall x_0 \in D : f $ differenzierbar, dann ist $f$ diffbar auf $D$ und $f' : D \rightarrow L(V,W)$ ist Ableitung von $f$. + +\subsubsection*{Jacobimatrix} + +Sei $D \subseteq \R^l$ offen, $f : D \rightarrow \R^k$, $i \in \{1, ..., k\}$, $j \in \{1, ..., l\}$ und $f = (f_1 \hdots f_m)^T$, dann: + +$\partial f(x) = \begin{pmatrix} \partial_1 f_1(x) & \hdots & \partial_l f_1(x) \\ \vdots & & \vdots \\ \partial_1 f_k(x) & \hdots & \partial_l f_k(x) \end{pmatrix}$ + +\subsubsection*{Gradient} + +Wenn $D \subseteq \R^m$ offen und $f : D \rightarrow \R$ bei $x \in D$ differenzierbar ist, dann: + +$\nabla f(x) := \begin{pmatrix} \partial_1 f(x) \\ \vdots \\ \partial_m f(x) \end{pmatrix} = f'(x)^T \in \R^m$ + +Identifiziert kritische Stellen mit $\nabla f(x) = 0$. + +\subsection*{Richtungsableitung} + +Sei $D \subseteq \R^m$ offen, $f : D \rightarrow \R$ und $v \in \R^m \setminus \{0\}$, dann ist Ableitung von $f$ bei $x$ in Richtung $v$: + +$\partial_v f(x) = \frac{\partial f}{\partial v}(x) := \displaystyle\lim_{t \to 0} \frac{1}{t} (f(x+tv)-f(x))$ + +Insofern Grenzwert in $\R$ existiert. Weiterhin gilt: + +$\partial_v f(x) = (\nabla f(x) | v)$ für $f \in C^1(D, \R)$. + +\subsubsection*{Hessematrix} + +Wenn $D \subseteq \R^m$ offen und $f \in C^n(D, \R)$, dann $\nabla f \in C^1(D,\R^m)$ und somit: + +$\nabla^2 f(x) = \begin{pmatrix} \partial_1 \partial_1 f(x) & \hdots & \partial_m \partial_1 f(x) \\ \vdots & & \vdots \\ \partial_1 \partial_m f(x) & \hdots & \partial_m \partial_m f(x) \end{pmatrix}$ + + Diese Matrix ist nach dem Theorem von Schwarz, welches ebendies besagt, symmetrisch. Laplaceop. $\Delta f(x) = \partial_{11}f(x) + ... + \partial_{mm} f(x)$ ist Spur. + +\subsection*{Taylor in mehreren Dimensionen} + +Sei $D \subseteq \R^l$ offen, $f \in C^{n+1}(D, \R)$, $x \in D$, $r > 0$ mit $\overline B(x,r) \subseteq D$ und $h \in \R^l$ mit $|h|_2 < r$, dann: + +$T_{n,x} f(x+h) = f(x) + \displaystyle\sum_{j=1}^n \frac{1}{j!} \displaystyle\sum_{\alpha_1, ..., \alpha_j=1}^l h_{\alpha_1} \cdot \hdots \cdot h_{\alpha_j} ( \partial_{\alpha_1} \hdots \partial_{\alpha_j} f )(x)$ + +Dies bedeutet insbesondere für $n=2$: + +$f(x+h) = f(x) + (\nabla f(x)|h) + \frac{1}{2}(\nabla^2 f(x)h|h) + O(|h|_2^3)$ + +\subsubsection*{Restglied} + +$R_{n,x} f(x+h) = f(x+h) - T_{n,x}f(x+h)$ + +\subsection*{Definitheit} + +Sei $A \in L(\R^m)$ symmetrisch, dann ist $A$: + +\begin{description}[leftmargin=!,labelwidth=28mm] + \item[positiv definit] $\forall v \in \R^m \setminus \{0\} : (Av|v) > 0$ + \item[negativ definit] $\forall v \in \R^m \setminus \{0\} : (Av|v) > 0$ + \item[positiv semidefinit] $\forall v \in \R^m : (Av|v) \geq 0$ + \item[negativ semidefinit] $\forall v \in \R^m : (Av|v) \leq 0$ +\end{description} + +\subsubsection*{Definitheit von $2\times 2$ Hessematrizen} + +Sei $A = \begin{pmatrix} a & b \\ b & d \end{pmatrix} \in \R^{2 \times 2}$ eine symmetrische Matrix: + +\begin{enumerate}[label=(\alph*)] + \item positiv definit $\Leftrightarrow$ $a > 0, ad > b^2$ + \item negativ definit $\Leftrightarrow$ $a < 0, ad > b^2$ + \item positiv semidefinit $\Leftrightarrow$ $a \geq 0, d \geq 0, ad \geq b^2$ + \item negativ semidefinit $\Leftrightarrow$ $a \leq 0, d \leq 0, ad \geq b^2$ + \item indefinit $\Leftrightarrow ad < b^2$ +\end{enumerate} + +\subsubsection*{Definitheit über Eigenwerte} + +\begin{enumerate}[label=(\alph*)] + \item positiv definit $\Leftrightarrow \forall \lambda \in Spec(A) : \lambda > 0$ + \item negativ definit $\Leftrightarrow \forall \lambda \in Spec(A) : \lambda < 0$ + \item positiv semidefinit $\Leftrightarrow \forall \lambda \in Spec(A) : \lambda \geq 0$ + \item negativ semidefinit $\Leftrightarrow \forall \lambda \in Spec(A) : \lambda \leq 0$ +\end{enumerate} + +\subsection*{Extremstellen} + +Seien $D \subseteq \R^m$ offen, $f \in C^2(D, \R)$, $x \in D$, dann: + +\begin{description}[leftmargin=!,labelwidth=28mm] + \item[Maximum] $\nabla^2 f(x)$ negativ semidefinit + \item[Minimum] $\nabla^2 f(x)$ positiv semidefinit +\end{description} + +Wenn $\nabla f(x) = 0$: + +\begin{description}[leftmargin=!,labelwidth=28mm] + \item[Maximum (strikt)] $\nabla^2 f(x)$ negativ definit + \item[Minimum (strikt)] $\nabla^2 f(x)$ positiv definit +\end{description} + +\subsection*{Umkehrsatz} + +Seien $D \subseteq \R^m$ offen, $f \in C^1(D, \R^m)$, $x_0 \in D$, $y_0 = f(x_0)$, $f'(x_0) \in L(\R^m$ bijektiv, dann: + +$\exists U \subseteq D \text{ offen }, V \subseteq \R^m : x_0 \in U, y_0 \in V, f_U : U \rightarrow V \text{ bijektiv }, V \subseteq f(D), (f_U)^{-1} \in C^1(V, U), \forall x \in U : f'(x) \text{ invertierbar}$. Insbesondere: + +$\forall y=f(x) \in V : (f_U^{-1})'(y) = f'(f_U^{-1}(y))^{-1} = f'(x)^{-1}$ + +\subsubsection*{Diffeomorphismen} + +Seien $D \subseteq \R^m$ offen, $f \in C^1(D, \R^m)$, $\tilde D = f(D)$, $f$ injektiv, $\forall x \in D: f'(x)$ invertierbar. Dann ist $\tilde D$ offen und $f : D \rightarrow \tilde D$ ein Diffeomorphismus, d.h. $D$, $\tilde D$ offen, $f$ bijektiv und $f \in C^1(D, \R^m)$, $f^{-1} \in C^1(\tilde D, \R^m)$ + +\subsubsection*{Polarkoordinaten} + +$\phi : D = (\R \setminus \{0\}) \times \R \rightarrow \R^2; (r, \varphi) \mapsto \begin{pmatrix} r \cos \varphi \\ r \sin \varphi \end{pmatrix} = \begin{pmatrix} x \\ y \end{pmatrix}$ + +\subsection*{Satz über implizit definierte Funktionen} + +Seien $D \subseteq \R^{m+k}$ offen, $f \in C^1(D, \R^k)$, $(x_0, y_0) \in D$ mit $f(x_0, y_0) = 0$ und $(\partial_y f)(x_0, y_0) \in L(\R^k)$ bijektiv. Dann $\exists \text{ offene } U_x \subseteq \R^m, U_y \subseteq \R^k$ sowie Abbildung $\varphi \in C^1(U, \R^k)$ mit: + +\begin{enumerate}[label=(\alph*)] + \item $(x_0, y_0) \in U_x \times U_y \subseteq D$, $\varphi(U_x) \subseteq U_y$, $\varphi(x_0)=y_0$, $\forall (x, y) \in U_x \times U_y : \partial_y f(x,y) \in L(\R^k)$ bijektiv + \item $(x, y) \in U_x \times U_y \land f(x,y)=0 \\ \Leftrightarrow x \in U_x \land y=\varphi(x)$ + \item $\varphi'(x) = -[(\partial_y f)(x, \varphi(x))]^{-1}(\partial_x f)(x, \varphi(x))$ +\end{enumerate} + +Insbesondere auch $\forall x \in U_x : f(x, \varphi(x))=0$ + +\subsection*{$C^1$-Flächen} + +$M \subseteq \R^3$ ist eingebettete $C^1$-Fläche, wenn $\forall p \in M \exists \text{ offene } V, U \subseteq \R^3 : p \in V \land \psi : V \rightarrow U$ so, dass $\psi(V \cap M) = U \cap (\R^2 \times \{0\})$. $\psi$ heißt dann Karte. + +\subsubsection*{Charakterisierung} + +\begin{enumerate}[label=(\alph*)] + \item $M$ ist $C^1$-Fläche + \item $\forall p \in M \exists \text{ offenes } D \subseteq \R^3 \land g \in C^1(D, \R) : \\ p \in D \land \forall w \in D : \nabla g(w) \neq 0 \land M \cap D = \{w\in D | g(w) = 0\}$ (lokale Nullstellenmenge) + \item $\forall p \in M \exists i < j \in \{1, 2, 3\}, \text{ offene } U_1 \subseteq \R^1, U_2 \subseteq \R^2, h \in C^1(U_2, U_1) : p_k \in U_1, (p_i, p_j) \in U_2$ mit $\{k\} = \{1, 2, 3\} \setminus \{j, j\}$, $h(U_2) \subseteq U_1$ und für $Z=\{x \in \R^3 | (x_i, x_j) \in U_2, x_k \in U_1\}$ gilt $M \cap Z = \{x \in \R^3 | (x_i, x_j) \in U_2, x_k = h(x_i, x_j)\}$ (lokaler Graph) + \item $\forall p \in M \exists \text{ offenes } U_0 \subseteq \R^2, W \subseteq \R^3, F \in C^1(U_0, \R^3) : p \in W, \forall (s, t) \in U_0 : Rg(F'(s, t)) = 2$ und $F: U_0 \rightarrow M \cap W$ bijektiv mit stetiger Umkehrabbildung. (lokale Parameterisierung) +\end{enumerate} + +\subsubsection*{Tangentialraum} + +Tangentialraum an $m$ bei $p=F(u_0)$ wobei $F$ lokale Parametrisierung: + +$T_p M = F'(u_0)(\R^2) = \left\{ F'(u_0) \begin{pmatrix} s \\ t \end{pmatrix} \middle| s, t \in \R \right\}$ + +\subsubsection*{Normalenraum} + +Normalenraum an $M$ bei $p$ und $g(p)=0$ wobei $g$ lokale Nullstellenmenge: + +$N_p M = lin\{\nabla g(p)\} = \{t\nabla g(p) | t \in \R\}$ + +\subsection*{Lagrange} + +Seien $D \subseteq \R^l$ offen, $f \in C^1(D, \R$ und $g \in C^1(D, \R^k)$ mit $1 \leq k \leq l$. Setze $M=\{z\in D| g(z)=0\}$. $f$ besitze auf $M$ ein Extremum bei $z_0 \in M$ und $g'(z_0)$ habe Rang $k$. Dann existieren Lagrangesche Multiplikatoren $\lambda_1, ..., \lambda_k \in \R$ so, dass: + +$\nabla f(z_0) = \lambda_1\nabla g_1(z_0) + \hdots + \lambda_k\nabla g_k(z_0)$ + +Ferner gilt $g_1(z_0) = 0, \hdots, g_k(z_0)=0$. + +\section*{Kurvenintegrale} + +Sei $\gamma \in C([a, b], \R^m)$ ein Weg und $\Gamma = \gamma([a, b])$ die dazugehörige Kurve. Die Länge von $\Gamma$ wird durch Polygonzüge approximiert. Sei dazu $a \leq b$ und $\mathcal{Z}(a, b)$ die Menge aller Zerlegungen $Z = \{a = t_0, t_1, ..., t_{n-1}, t_n = b\}$. Für $Z \in \mathcal{Z}$ wird gesetzt: + +$l(\gamma, Z) = \sum_{j=1}^n |\gamma(t_j) - \gamma(t_{j-1})|_2$ + +\subsection*{Rektifizierbarkeit} + +$\gamma : [a, b] \rightarrow \R^m$ ist rektifizierbar, wenn: + +$l_{[a, b]}(\gamma) = ||\gamma||_{BV} := \sup_{Z\in \mathcal{Z}} l(\gamma, Z) < \infty$ + +$l(\gamma)$ ist dann die Länge von $\gamma$. + +\subsubsection*{Wegintegral} + +Sei $\gamma \in C^1([a, b], \R^m)$, dann ist $\gamma$ rektifizierbar mit: + +$l(\gamma) = \int_a^b |\gamma'(t)|_2 dt$ + +\subsection*{Kurvenintegrale und Potentiale} + +Sei $\gamma \in C([a, b], \R^m)$ stückweise $C^1$, $\Gamma = \gamma([a, b])$. + +\subsubsection*{Kurvenintegral erster Art} + +Sei reelles $f \in C(\Gamma, \R)$ gegeben: + +\vspace*{-5mm} +$$\int_\Gamma f d\gamma = \int_\Gamma f(x) d\gamma := \int_a^b f(\gamma(t)) | \gamma'(t) |_2 dt$$ + +\subsubsection*{Kurvenintegral zweiter Art} + +Sei vektorwertiges $F \in C(\Gamma, \R^m)$ gegeben: + +\vspace*{-5mm} +$$\int_\Gamma F \cdot dx = \int_\Gamma F(x) \cdot dx := \int_a^b (F(\gamma(t))|\gamma'(t)) dt$$ + +\subsubsection*{Wegunabhängigkeit} + +Sei $D \subseteq \R^m$ offen, dann ist $F \in C(D, \R^m)$ wegunabhängig auf $D$, wenn für alle stückweisen $C^1$-Kurven $\gamma_1, \gamma_2 \in C([a, b], \R^m)$ in $D$ mit gleichem Anfangs- und Endpunkt gilt: + +$$\int_{\Gamma_1} F \cdot dx = \int_{\Gamma_2} F \cdot dx$$ + +Ein $\phi \in C^1(D, \R)$ heißt Potential von $F$ auf $D$, wenn $\nabla\phi = F$ auf $D$. $F$ ist dann Gradientenfeld. + +Weiterhin ist $F$ wegunabhängig auf $D$ gdw. $F$ ein Potential $\phi$ auf $D$ hat. + +Ferner gilt dann: $\int_\Gamma F \cdot dx = \phi(\gamma(b))-\phi(\gamma(a))$ + +\subsubsection*{Poincar\'e} + +Sei $D \subseteq \R^m$ offen und sternförmig und $F \in C^1(D, \R^m)$ sei rektifizierbar. Dann hat $F$ ein Potential auf $D$, insbesondere auch auf jeder Kugel $B(x_0, r) \subseteq D$. + +\section*{Gewöhnliche Differentialgleichung} + +\subsection*{Lokale Lipschitzstetigkeit} + +Sei $M \subseteq \R^m$ und $J$ ein Intervall. $g : J \times M \rightarrow \R^k$ ist lokal Lipschitz in $x$, wenn: + +$\forall (t_0, x_0) \in J \times M \exists \delta = \delta(t_0, x_0) > 0, r = r(t_0, x_0) > 0, L = L(t_0, x_0) \geq 0 \forall t \in [t_0 - \delta, t_0 + \delta] \cap J \forall x, y \in \overline B(x_0, r) \cap M : |g(t,x) - g(t, y)|_2 \leq L|x-y|_2$. + +$f$ Lipschitz $\Rightarrow f$ lokal Lipschitz $\Rightarrow f$ stetig. + +$f$ stetig differenzierbar $\Rightarrow f$ lokal Lipschitz. + +Ein diff. $f$ ist Lipschitz gdw. $\partial_x^{(1)} f$ beschränkt ist. + +\subsection*{Anfangswertprobleme} + +Seien $J \subseteq \R$ ein Intervall, $t_0 \in J$ mit $t_0 < \sup J$, $D \subseteq \R^m$ offen, $f \in C(J \times D, \R^m)$ und $u_0 \in D$. + +\vspace*{-4mm} +\begin{align*} + u'(t) &= f(t, u(t)), t\geq t_0, t\in J \\ + u(t_0) &= u_0 +\end{align*} + +Für das Anfangswertproblem wird ein $t_1 \in J$ mit $t_1 > t_0$ und eine eindeutige Lösung $u \in C^1([t_0, t_1], \R^m)$ auf $[t_0, t_1]$ gesucht. + +\subsubsection*{Lokale Lipschitzstetigkeit im Kontext} + +Sei $f \in C(J \times D, \R^k)$, $D \subseteq \R^m$ offen, $J$ ein Intervall und es existieren alle partiellen Ableitungen $\frac{\partial}{\partial x_j} f \in C(J \times D, \R^k)$ für $j \in \{1, \hdots, m\}$. + +Dann ist $f$ lokal Lipschitz in $x$. + +\subsubsection*{Picard-Lindelöf (lokal)} + +Seien $J$ ein Intervall, $D \subseteq \R^m$ offen, $f \in C(J \times D, \R^m)$ lokal Lipschitz in $x$, $u_0 \in D$, $t_0 \in J$ mit $t_0 < \sup J$. Dann gelten: + +\begin{enumerate}[label=(\alph*)] + \item $\exists t_1 = t_1(u_0) > t_0 $ mit $t_1 \in J$ und eine eindeutige Lösung $u$ auf $[t_0, t_1]$ von $u'(t) = f(t, u(t))$ mit $u(t_0) = u_0$ + \item $u'(t) = f(t, u(t))$ mit $u(t_0) = u_0$ besitze zwei Lösungen $v_1$ und $v_2$ auf Intervallen $[t_0, T_1] \subseteq J$ bzw. $[t_0, T_2] \subseteq J$. Dann stimmen $v_1$ und $v_2$ auf $[t_0, T_3]$ mit $T_3 = \min\{T_1, T_2\}$ überein. +\end{enumerate} + +\subsubsection*{Picard-Lindelöf (maximal)} + +Unter den Voraussetzungen von Picard-Lindelöf (lokal) sei $u_0 \in D$, dann gilt weiterhin: + +\begin{enumerate}[label=(\alph*)] + \item $\exists$ maximale Existenzzeit $\overline t(u_0) \in (t_0, \sup J]$ und eine eindeutige maximale Lösung $u$ von $u'(t) = f(t, u(t))$ mit $u(t_0) = u_0$ auf $[t_0, \overline t(u_0))$ + \item Wenn $\overline t(u_0) < \sup J$, dann $\exists t_n \in (t_0, \overline t(u_0))$ mit $\lim_{n \to \infty} t_n = \overline t(u_0)$ so, dass die Blow-Up Bedingung erfüllt ist: $\lim_{n \to \infty} |u(t_n)|_n = \infty$ oder $\lim_{n \to \infty} \inf_{x \in \partial D} |u(t_n) - x|_2 = 0$ +\end{enumerate} + +\subsubsection*{Gronwallsche Ungleichung} + +Sei $J$ ein Intervall, $\varphi \in C(J)$, $\varphi \geq 0$, $t_0 = \min J$ und $a, b \geq 0$ Konstanten mit $\forall t \in J : 0 \leq \varphi(t) \leq a+b \int_{t_0}^t \varphi(s) ds$. Dann gilt: $\forall t \in J : \varphi(t) \leq a e^{b(t-t_0)}$ + +\subsubsection*{Obergrenze des Existenzintervall} + +Seien $J = [t_0, \infty)$, $D \subseteq \R^m$ offen, $M \subseteq \R^m$ abgeschlossen mit $M \subseteq D$, $f \in C(J \times D, \R^m)$ sei lokal Lipschitz in $x$, $u$ die maximale Lösung von $u'(t) = f(t, u(t))$ auf $[t_0, t(u_0))$ und es gelte $\forall t_0 \leq t \leq \overline t(u_0) : u(t) \in M$. Dann gelten: + +\begin{enumerate}[label=(\alph*)] + \item $\forall b > t_0 \exists c(b) \geq 0 : \forall t \in [t_0, b], x \in M : (f(t,x)|x) \leq c(b) (1+|x|_2^2) \Rightarrow \overline t(u_0) = \infty$ + \item Sei speziell $f: D \rightarrow \R^n$ und $\exists c \geq 0$ mit $\forall x \in M : (f(x)|x) \leq c(1+|x|_2)$, dann $\overline t(u_0) = \infty$ +\end{enumerate} + +Bedingung (a) folgt aus: $\forall b > t_0 \exists \tilde c(b) \geq 0 \\ \hspace*{4mm} \forall t \in [t_0, b], x \in M : |f(t,x)|_2 \leq \tilde c(b)(1+|x|_2)$ + +Bedingung (b) folgt aus: $\exists \tilde c > 0 \forall x \in M : \\ \hspace*{4mm} |f(x)|_2 \leq \tilde c(1+|x|_2)$ + +\subsubsection*{Formulierung als Problem 1. Ordnung} + +Jedes Anfangswertproblem $k$-ter Ordnung lässt sich in ein Problem 1. Ordnung umschreiben. + +Beispielsweise: Das Problem 2. Ordnung $u''(t)=h(t)-u(t)+u'(t)^2$ mit $u(0)=u_0$ und $u'(0)=u_1$ sowie $h \in C(\R, \R)$ wird formuliert als Problem 1. Ordnung: + +$$\begin{pmatrix}u(t)\\u'(t)\end{pmatrix}' = \begin{pmatrix}u'(t)\\u''(t)=h(t)-u(t)+u'(t)^2\end{pmatrix}$$ + +Sei $v_0(t):=u(t)$, $v_1(t):=u'(t)$ und $v(t):=\begin{pmatrix}v_0(t)\\v_1(t)\end{pmatrix}$ + +\vspace*{-4mm} +\begin{align*} + g : \R^3 &\rightarrow \R^2 \\ + \begin{pmatrix}t\\v_0\\v_1\end{pmatrix} &\mapsto \begin{pmatrix}v_1\\h(t)-v_0+v_1^2\end{pmatrix} +\end{align*} + +Insgesamt also: + +\vspace*{-4mm} +\begin{align*} + v'(t)&=g(t,v(t))=\begin{pmatrix}v_1(t)\\h(t)-v_0(t)+v_1(t)^2\end{pmatrix}\\ + v(0)&=\begin{pmatrix}u_0\\u_1\end{pmatrix} +\end{align*} + +\subsubsection*{Trennung der Variablen} + +Sei $u'(t)=g(t)h(u(t))$ mit $u(t_0)=u_0$ Anfangswertproblem mit $g \in C(\R, \R)$, $h \in C((a, b), \R)$, $u_0 \in (a, b)$ und $h(u_0) \neq 0$. $u$ ist Lösung, wenn $J$ Intervall mit $\forall t \in J : u(t) \in (a, b)$, $u \in C^1(J, \R)$ und $t_0 \in J$. + +\vspace*{-5mm} +$$u \text{ ist Lösung } \Rightarrow \int_{t_0}^t g(s) ds = \int_{u_0}^{u(t)} \frac{1}{h(x)} dx$$ +\vspace*{-3mm} + +Dies kann manchmal nach $u$ aufgelöst werden. diff --git a/content/analysis_3.tex b/content/analysis_3.tex new file mode 100644 index 0000000..483a52f --- /dev/null +++ b/content/analysis_3.tex @@ -0,0 +1,188 @@ +\section*{Nützliches aus der Mengenlehre} + +\subsection*{De Morgansche Regeln} + +Sei $\mathcal{B}$ ein Mengensystem. + +$$\left(\bigcup_{B\in \mathcal{B}} B \right)^c = \bigcap_{B\in \mathcal{B}} B^c \hspace*{8mm} \left(\bigcap_{B\in \mathcal{B}} \right)^c = \bigcup_{B\in \mathcal{B}} B^c$$ + +\subsection*{Mengen-Ring} + +Ein Mengensystem $\mathcal{A}$ ist ein Ring gdw. $\forall A, B \in \mathcal{A}$: + +\begin{enumerate}[label=(\alph*)] + \item $\emptyset \in \mathcal{A}$ + \item $B\setminus A \in \mathcal{A}$ + \item $A \cup B \in \mathcal{A}$ +\end{enumerate} + +\section*{$\sigma$-Algebren} + +Ein Mengensystem $\mathcal{A} \subseteq \mathcal{P}(X)$ ist $\sigma$-Algebra auf der nichtleeren Menge $X$ gdw.: + +\begin{enumerate}[label=(\alph*)] + \item $X \in \mathcal{A}$ + \item $A \in \mathcal{A} \Rightarrow A^c := X\setminus A \in \mathcal{A}$ + \item $\forall j \in \N : A_j \in \mathcal{A} \Rightarrow \bigcup_{j\in \N} A_j \in \mathcal{A}$ +\end{enumerate} + +\subsection*{Eigenschaften von $\sigma$-Algebren} + +Seien $\mathcal{A}$ eine $\sigma$-Algebra auf $X$, $n \in \N$, $\forall j \in \N : A_j \in \mathcal{A}$, dann ist $\mathcal{A}$ nach den folgenden Eigenschaften abgeschlossen unter abzählbaren Mengenoperationen: + +\begin{enumerate}[label=(\alph*)] + \item $\emptyset = X^c \in \mathcal{A}$ + \item $A_1 \bigcup \cdots \bigcup A_n \in \mathcal{A}$ + \item $A_1 \bigcap \cdots \bigcap A_n \in \mathcal{A}$ + \item $\bigcap_{j\in \N} A_j \in \mathcal{A}$ + \item $A_1 \setminus A_2 := A_1 \bigcap A_2^c \in \mathcal{A}$ +\end{enumerate} + +\subsection*{Erzeugte $\sigma$-Algebren} + +Die durch das nichtleere Mengensystem $\mathcal{E} \subseteq \mathcal{P}(X)$ auf $X$ erzeugte $\sigma$-Algebra ist wie folgt definiert: + +\vspace*{-4mm} +$$\sigma(\mathcal{E}) := \bigcap\{ \mathcal{A} \subseteq \mathcal{P}(X) | \mathcal{A} \text{ ist } \sigma \text{-Algebra}, \mathcal{E} \subseteq \mathcal{A} \}$$ + +Der Erzeuger $\mathcal{E}$ ist hierbei allg. nicht eindeutig. + +\subsubsection*{Eigenschaften erzeugter $\sigma$-Algebren} + +Sei $\emptyset \neq \mathcal{E} \subseteq \mathcal{P}(X)$, dann gilt: + +\begin{enumerate}[label=(\alph*)] + \item $\mathcal{A}$ ist $\sigma$-Algebra $\land$ $\mathcal{E} \subseteq \mathcal{A} \Rightarrow \mathcal{E} \subseteq \sigma(\mathcal{E}) \subseteq \mathcal{A}$ + \item $\sigma(\mathcal{E})$ ist kleinste $\mathcal{E}$ enthaltende $\sigma$-Algebra. + \item $\mathcal{E}$ ist $\sigma$-Algebra $\Rightarrow \mathcal{E} = \sigma(\mathcal{E})$ + \item $\mathcal{E} \subseteq \mathcal{E}' \subseteq \mathcal{P}(X) \Rightarrow \sigma(\mathcal{E}) \subseteq \sigma(\mathcal{E}')$ +\end{enumerate} + +\subsection*{Borelsche $\sigma$-Algebra} + +Sei $X$ ein metrischer Raum und $\mathcal{O}(X)$ das System der in $X$ offenen Mengen, dann ist $\mathcal{B}(X) := \sigma(\mathcal{O}(X))$ die Borelsche $\sigma$-Algebra auf $X$. + +Im Speziellen wird $\mathcal{B}_m := \mathcal{B}(\R^m)$ gesetzt. + +$\mathcal{B}_m$ enthält insb. alle offenen und abgeschlossenen Mengen in $\R^m$ sowie deren abzählbaren Vereinigungen und Durchschnitte. + +\subsubsection*{Charakterisierung} + +\vspace*{-4mm} +\begin{align*} + \mathcal{B}_m &= \sigma(\{(a, b) | a, b \in \mathbb{Q}^m, a \leq b\}) \\ + &= \sigma(\{(a, b] | a, b \in \mathbb{Q}^m, a \leq b\}) +\end{align*} + +\section*{Maße auf $\sigma$-Algebren} + +Sei $\mathcal{A}$ eine $\sigma$-Algebra auf $X$. + +$\mu : \mathcal{A} \rightarrow [0, \infty]$ ist positives Maß auf $\mathcal{A}$ gdw.: + +\begin{enumerate}[label=(\alph*)] + \item $\mu(\emptyset) = 0$ + \item $\forall \text{ disjunkte } \{A_j | j \in \N\} \subseteq \mathcal{A} :\\ \hspace*{4mm} \mu(\dot\bigcup_{j\in \N} A_j) = \sum_{j\in \N} \mu(A_j)$ +\end{enumerate} + +\subsection*{Maßraum} + +Ein Tripel $(X, \mathcal{A}, \mu)$ ist Maßraum. Ein endlicher Maßraum erfüllt zusätzlich $\mu(X) < \infty$. + +Ein Wahrscheinlichkeitsmaß erfüllt $\mu(X) = 1$. + +\subsection*{Punkt- / Diracmaß} + +Für fest gewählte $\mathcal{A} = \mathcal{P}(X)$, $x \in X$ ist ein Wahrscheinlichkeitsmaß für $A \subseteq X$ definiert: + +$$\delta_x(A) := \begin{cases} + 1 & x \in A \\ + 0 & x \notin A +\end{cases}$$ + +Dieses wird Punkt- / Diracmaß auf $\mathcal{A}$ genannt. + +\subsection*{Zählmaß} + +Sei $\mathcal{A} = \mathcal{P}(\N)$ und $\forall j \in \N : p_j \in [0, \infty]$ fest gewählt. + +$\mu(A) := \sum_{j\in A} p_j$ für $A \subseteq \N$ ist Maß auf $\mathcal{P}(\N)$. + +Gilt zusätzlich $\forall j \in \N : p_j = 1$ so heißt $\mu$ Zählmaß. + +\subsection*{Eigenschaften von Maßen} + +Sei $(X, \mathcal{A}, \mu)$ Maßraum und $A, B, A_j \in \mathcal{A}$ für $j \in \N$. + +\begin{description}[leftmargin=!,labelwidth=26mm] + \item[Monotonie] $A \subseteq B \Rightarrow \mu(A) \leq \mu(B)$ + \item[$\sigma$-Subadditivität] $\mu(\dot\bigcup_{j\in \N} A_j) \leq \sum_{j\in \N} \mu(A_j)$ + \item[Stetigkeit (unten)] $A_j \uparrow \Rightarrow \displaystyle\lim_{j\to \infty} \mu(A_j) = \mu(\bigcup_{j\in \N} A_j)$ + \item[Stetigkeit (oben)] $A_j \downarrow \land \hspace*{1mm} \mu(A_1) < \infty \\ \hspace*{4mm}\Rightarrow \displaystyle\lim_{j\to \infty} \mu(A_j) = \mu(\bigcap_{j\in \N} A_j)$ +\end{description} + +Für $\mu(A) < \infty$ folgt $\mu(B\setminus A) = \mu(B) - \mu(A)$. + +Für endliche Maße gilt insb. $\mu(A^c) = \mu(X) - \mu(A)$. + +\subsection*{Prämaß} + +Eine Abb. $f : \mathcal{A} \rightarrow [0, \infty)$ ist ein Prämaß auf Ring $\mathcal{A}$ gdw.: + +\begin{enumerate}[label=(\alph*)] + \item $\mu(\emptyset) = 0$ + \item $\{A_j | j \in \N\} \subseteq \mathcal{A}$ disjunkt und $A = \bigcup_{j\in \N} A_j \in \mathcal{A} \Rightarrow \mu(A) = \sum_{j\in \N} \mu(A_j)$ +\end{enumerate} + +\section*{Lebesguemaß} + +\subsection*{System der Intervalle} + +Sei $I = (a, b] \subseteq \R^m$ für $a, b \in \R^m$ mit $a \leq b$, dann wird das System von Intervallen $\mathcal{J}_m$ definiert: + +$\lambda(I) = \lambda_m(I) := (b_1 - a_1) \cdot \hdots \cdot (b_m - a_m)$ + +\subsection*{Ring der Figuren} + +$$\mathcal{F}_m = \left\{ A = \bigcup_{j=1}^n I_j | I_j \in \mathcal{J}_m, n \in \N \right\}$$ + +\subsubsection*{Eigenschaften} + +Seien $I_1, I_2 \in \mathcal{J}_m$: + +\begin{enumerate}[label=(\alph*)] + \item $\sigma(\mathcal{F}_m) = \mathcal{B}_m$ + \item $I_1 \cap I_2 \in \mathcal{J}_m$ + \item $I_1 \setminus I_2 \in \mathcal{F}_m$ sowie endliche Vereinigung disjunkter Intervalle aus $\mathcal{J}_m$ + \item $\forall A \in \mathcal{F}_m: A$ ist endliche Vereinigung disjunkter Intervalle aus $\mathcal{J}_m$ + \item $\mathcal{F}_m$ ist Ring +\end{enumerate} + +\section*{Messbare Funktionen} + +Sei $\mathcal{A}$ eine $\sigma$-Algebra auf $X\neq \emptyset$ und $\mathcal{B}$ eine $\sigma$-Algebra auf $Y\neq \emptyset$ sowie $f : X \rightarrow Y$ Funktion. + +$f$ heißt ($\mathcal{A}$-$\mathcal{B}$-)messbar gdw. $\forall B \in \mathcal{B} : f^{-1}(B) \in \mathcal{A}$ + +\subsection*{Borel-Messbarkeit} + +Seien $X, Y$ metrische Räume. + +Die Funktion $f : X \rightarrow Y$ heißt Borel-messbar, wenn sie $\mathcal{B}(X)$-$\mathcal{B}(Y)$-messbar ist. + +\subsection*{Eigenschaften} + +Seien $\mathcal{A}, \mathcal{B}, \mathcal{C}$ $\sigma$-Algebren auf $X, Y, Z \neq \emptyset$. + +\begin{enumerate}[label=(\alph*)] + \item $f : X \rightarrow Y$ ist $\mathcal{A}$-$\mathcal{B}$-mb., $g : Y \rightarrow Z$ ist $\mathcal{B}$-$\mathcal{C}$-mb. $\Rightarrow g \circ f : X \rightarrow Z$ ist $\mathcal{A}$-$\mathcal{C}$-mb. + \item $\emptyset \neq \mathcal{E} \subseteq \mathcal{P}(Y)$, $\mathcal{B} = \sigma(\mathcal{E})$, $f: X \rightarrow Y$ dann ist $f$ messbar gdw. $\forall E \in \mathcal{E} : f^{-1}(E) \in \mathcal{A}$ + \item $X, Y$ metrische Räume, $f : X \rightarrow Y$ stetig $\Rightarrow f$ ist Borel-messbar + \item $f : X \rightarrow \R^m$ ist $\mathcal{A}$-$\mathcal{B}_m$-mb. gdw. $\forall i \in \{1, \dots, m\} : f_i : X \rightarrow \R$ ist $\mathcal{A}$-$\mathcal{B}_1$-mb. + \item $f, g$ sind $\mathcal{A}$-$\mathcal{B}_1$-mb. und $\alpha, \beta \in \R \Rightarrow fg : X \rightarrow \R$ und $\frac{1}{f} : \{x \in X | f(x) \neq 0\} \rightarrow \R$ mb. + \item $f : X \rightarrow \R^m$ ist $\mathcal{A}$-$\mathcal{B}_m$-mb. $\\\Rightarrow g : X \rightarrow \R; x \mapsto |f(x)|_2$ ist $\mathcal{A}$-$\mathcal{B}_1$-mb. + \item $X = W \dot\cup Z$ mit $\emptyset \neq W, Z \in \mathcal{A}$, $f : W \rightarrow Y$ ist $\mathcal{A}_W$-$\mathcal{B}$-mb., $g : Z \rightarrow Y$ ist $\mathcal{A}_Z$-$\mathcal{B}$-mb. $\Rightarrow h(x) = \begin{cases} + f(x) & x \in W \\ + g(x) & x \in Z +\end{cases}$ ist $\mathcal{A}$-$\mathcal{B}$-mb. +\end{enumerate} diff --git a/content/lineare_algebra.tex b/content/lineare_algebra.tex new file mode 100644 index 0000000..a92a3a8 --- /dev/null +++ b/content/lineare_algebra.tex @@ -0,0 +1,706 @@ +\section*{Relationen} + +Sei $R := A \times B$ eine Relation. + +\subsection*{Linkstotal} + +$\forall a \in A \exists b \in B : (a,b) \in R$ + +\subsection*{Rechtstotal / Surjektiv} + +$\forall b \in B \exists a \in A : (a,b) \in R \text{ bzw. } f(a)=b$ + +\subsection*{Linkseindeutig / Injektiv} + +$\forall a_1, a_2 \in A \forall b \in B :$ \newline +\hspace*{5mm} $(a_1,b) \in R \land (a_2,b) \in R \implies a_1=a_2$ + +\subsection*{Rechtseindeutig} + +$\forall a \in A \forall b_1, b_2 \in B:$ \newline +\hspace*{5mm} $(a,b_1) \in R \land (a,b_2) \in R \implies b_1=b_2$ + +\subsection*{Eigenschaften von Relationen} + +\begin{description}[leftmargin=!,labelwidth=25mm] + \item[reflexiv] $\forall x \in M : (x, x) \in R$ + \item[symmetrisch] $\forall x, y \in M : xRy \Leftrightarrow yRx$ + \item[antisymmetrisch] $\forall x, y \in M : xRy \land yRx \Rightarrow x=y$ + \item[transitiv] $\forall x, y, z \in M : xRy \land yRz \Rightarrow xRz$ +\end{description} + +\subsection*{Äquivalenzrelationen} + +Eine Relation $R$ auf Menge $M$ ist Äquivalenzrelation, wenn sie reflexiv, symmetrisch und transitiv ist. + +\section*{Gruppen} + +$\star : M \times M \rightarrow M$ ist Verknüpfung auf Menge $M$ abhängig der Argument-Reihenfolge. Das Tupel $(M, \star)$ ist Gruppe, wenn: + +\begin{enumerate}[label=(\alph*)] + \item $\star$ ist assoziativ + \item $\exists e \in M \forall x \in M : x \star e = e \star x = x$ + \item $\forall x \in M \exists y \in M : x \star y = y \star x = e$ +\end{enumerate} + +Ist $\star$ kommutativ, dann $(M, \star)$ abelsche Gruppe. + +\subsection*{Assoziativität} + +$\forall m_1, m_2, m_3 \in M : (m_1 \star m_2) \star m_3 = m_1 \star (m_2 \star m_3)$ + +\subsection*{Kommutativität} + +$\forall m_1, m_2 \in M : m_1 \star m_2 = m_2 \star m_1$ + +\subsection*{Untergruppen} + +$(M, \star)$ ist Gruppe. $(H, \circ)$ ist Untergruppe, wenn: + +\begin{enumerate}[label=(\alph*)] + \item $H \subseteq M$ + \item $(H, \circ)$ ist Gruppe + \item $\forall h_1, h_2 \in H : h_1 \circ h_2 = h_1 \star h_2$ +\end{enumerate} + +\subsubsection*{Untergruppenkriterium} + +$H \subseteq G$ ist Untergruppe von $G$ wenn: + +$H \neq \emptyset \land \forall h_1, h_2 \in H : h_1 \star h_2^{-1} \in H$ + +\subsection*{Gruppenhomomorphismen} + +Seien $(G, \star)$ und $(H, \circ)$ Gruppen. $f: G \rightarrow H$ ist Gruppenhomomorphismus wenn: + +$\forall g_1, g_2 \in G: f(g_1 \star g_2) = f(g_1) \circ f(g_2)$ + +\subsubsection*{Eigenschaften von Homomorphismen} + +\begin{enumerate}[label=(\alph*)] + \item $f(e_G) = e_H$ + \item $\forall g \in G : f(g^{-1}) = f(g)^{-1}$ + \item $f(G)$ ist Untergruppe von $H$ + \item $f \in Hom(G, H)$ ist genau dann injektiv, wenn $Kern(f) = \{e_G\}$ +\end{enumerate} + +\subsection*{Weitere Homomorphismen} + +\begin{enumerate}[label=(\alph*)] + \item $f : G \rightarrow G$ ist Endomorphismus + \item Bijektives $f: G \rightarrow H$ ist Isomorphismus + \item Bijektives $f \in End(V)$ ist Automorphismus +\end{enumerate} + +\section*{Ringe} + +$(R, +, *)$ ist Ring, wenn: + +\begin{enumerate}[label=(\alph*)] + \item $(R, +)$ ist abelsche Gruppe + \item $*$ ist assoziativ + \item $\forall x \in R : 1_R * x = x * 1_R = x$ + \item $x*(y+z) = (x*y)+(x*z)$ + \item $(y+z)*x = (y*x)+(z*x)$ +\end{enumerate} + +Ist $*$ kommutativ, $(R, +, *)$ ein kommutativer Ring. + +\subsection*{Teilringe} + +Unter $+$ und $*$ geschlossene Teilmenge $T \subseteq R$ ist Teilring von $R$. + +\subsection*{Ringhomomorphismen} + +$\phi : R \rightarrow S$ ist Ringhomomorphismus, wenn: + +\begin{enumerate}[label=(\alph*)] + \item $\forall x, y \in R : \phi(x +_R y) = \phi(x) +_S \phi(y)$ + \item $\forall x, y \in R : \phi(x *_R y) = \phi(x) *_S \phi(y)$ + \item $\phi(1_R) = 1_S$ +\end{enumerate} + +\section*{Körper} + +Ein Körper ist kommutativer Ring $K$ mit $0_K \neq 1_K$ und für den jedes $x \neq 0_K$ invertierbar ist. + +\section*{Matrizen} + +\subsection*{Invertierbare Matrizen} + +Für einen kommutativen Ring $R$ ist die "general linear Group": + +$GL_p(R) := \{ A \in R^{p \times p} | \exists B \in R^{p \times p} : AB = BA = I_p \}$ + +$A \in GL_p(R)$ sind invert. / reguläre Matrizen. + +\subsection*{Elementarmatrizen} + +$$R^{2 \times 3} \ni E_{2,3} = \begin{pmatrix} + 0 & 0 & 0 & 0 \\ + 0 & 0 & 1 & 0 +\end{pmatrix}$$ + +\subsection*{Äquivalenz von Matrizen} + +$\exists S \in GL_q(K), T \in GL_p(K) : B = T A S$ ($A, B \in K^{p \times q}$) + +\subsection*{Ähnlichkeit von Matrizen} + +$A, \tilde A \in K^{d \times d}$ ähnlich $\Leftrightarrow \exists S \in GL_d(K) : \tilde A = S^{-1}AS$ + +Ähnliche Matrizen haben die gleiche Determinante, Spur und Rang. + +\subsection*{Determinante} + +\begin{enumerate}[label=(\alph*)] + \item $det(A) \neq 0 \Leftrightarrow A \text{ ist invertierbar}$ + \item $det(A*B) = det(A) * det(B)$ + \item $det(A)^{-1} = det(A)^{-1}$ falls $A \in GL_n(K)$ + \item $det(A) = det(A^T)$ +\end{enumerate} + +\section*{Vektorräume} + +Ein $K$-Vektorraum ist kommutative Gruppe $(V, +)$ mit skalarer Multiplikation $* : K \times V \rightarrow V, (a, v) \mapsto a * v$ sowie: + +\begin{enumerate}[label=(\alph*)] + \item $\forall v \in V : 1_K * v = v$ + \item $\forall a, b \in K \forall v \in V : a*(b*v)=(a*b)*v$ + \item $\forall a, b \in K \forall u, v \in V : a*(u+v)=a*u+a*v$ + \item $\forall a, b \in K \forall u, v \in V : (a+b)*v=a*v+b*v$ +\end{enumerate} + +\subsection*{Untervektorräume} + +$K$-Untervektorraum $U$ von $V$ ist Teilmenge $U \subseteq V$ die bzgl. Addition Untergruppe von $V$ ist und für die gilt: $\forall a \in K, u \in U : a*u \in U$ (d.h. skalare Multiplikation geschlossen) + +\subsubsection*{Untervektorraumkriterium} + +Seien $K$ Körper, $V$ $K$-Vektorraum und $U \subseteq V$. Dann ist äquivalent: + +\begin{enumerate}[label=(\alph*)] + \item $U$ ist Untervektorraum von $V$ + \item $U \neq \emptyset$, $\forall u_1, u_2 \in U : u_1 + u_2 \in U$ und $\forall a \in K, u \in U : a*u \in U$ +\end{enumerate} + +\subsubsection*{$\phi$-invariante Unterräume} + +$U \subset V$ ist $\phi$-invariant, wenn $\phi(U) \subset U$. + +\subsubsection*{Summe} + +$U_1 + U_2 = \{u_1+u_2 | u_1 \in U_1, u_2 \in U_2 \}$ ist Summe von $U_1$ und $U_2$, kleinster UVR der $U_1 \cup U_2$ enthält. + +$dim(U_1 + U_2) = dim(U_1) + dim(U_2) - dim(U_1 \cap U_2)$ + +\subsubsection*{Direkte Summe} + +$U_1 + U_2$ ist direkte Summe, wenn $U_1 \cap U_2 = {0}$. + +d.h. gdw. der Schnitt eines UVR mit der Summe aller anderen UVR nur den Nullvektor enthält. + +$dim(U_1 \oplus U_2)=dim(U_1)+dim(U_2)$ + +\subsection*{Homomorphismen} + +Seien $V, W$ zwei $K$-Vektorräume. $\phi : V \rightarrow W$ ist Vektorraumhomomorphismus respektive $K$-lineare Abbildung, wenn: + +\begin{enumerate}[label=(\alph*)] + \item $\forall u, v \in V : \phi(u+v) = \phi(u)+\phi(v)$ + \item $\forall a \in K, v \in V : \phi(a*v) = a*\phi(v)$ +\end{enumerate} + +$Rang(\phi) := dim(Bild(\phi))$ mit $\phi \in Hom(V, W)$ + +\subsection*{Basen} + +Teilmenge $B \subseteq V$ ist Basis von $V$, wenn sich $\forall v \in V$ auf genau eine Art als Linearkombination von $B$ schreiben lässt. Jede Basis von $K^p$ hat genau $p$ Elemente. + +\subsubsection*{Lineare Unabhängigkeit} + +$\sum_{i=1}^{k} \lambda_i * v_i = 0 \Rightarrow \forall i \in \{1, \cdots, k\} : \lambda_i = 0$ + +\subsubsection*{Dimension} + +Ist Mächtigkeit der Basis, $dim(V) := |B|$ + +$U$ Untervektorraum $V$, dann: $dim(U) \leq dim(V)$ + +$dim(V) = dim(Kern(\phi)) + dim(Bild(\phi))$ + +\subsubsection*{Abbildungsmatrizen} + +$D_C(\phi(v)) = D_{CB}(\phi) * D_B(v)$ + +\subsubsection*{Basiswechselformel} + +Seien $V, W$ $K$-Vektorräume; $\phi \in Hom(V, W)$; $B, \tilde B$ Basen von $V$; $C, \tilde C$ Basen von W. Dann gilt: + +$D_{\tilde C \tilde B}(\phi) = D_{\tilde C C}(I_W) * D_{CB}(\phi) * D_{B\tilde B}(I_V)$ + +\subsection*{Dualräume} + +Sei $V$ ein $K$-Vektorraum. Die Menge aller linearen Abbildungen $V \rightarrow K$ ist der Dualraum: $V^* := \{f: V \rightarrow K | f \text{ ist linear}\}$. + +$f \in V^*$ werden als Linearformen bezeichnet. + +\subsubsection*{Duale Basis} + +Sei $B = \{b_1, \cdots, b_n\}$ Basis von $V$, dann ist $B^* = \{b_1^*, \cdots, b_n^*\}$ Basis des Dualraums $V^*$, also die zu $B$ von $V$ duale Basis. + +Die für $b_i$ eindeutige Abb. $b_i^* : V \rightarrow \K$ erfüllt $b_i^*(b_i) = 1$ und $b_i^*(b_j) = 0$ für $j\neq i$. + +\subsection*{Faktor- / Quotientenräume} + +$v_1 \thicksim v_2 \Leftrightarrow v_1 - v_2 \in U$ für $v_1, v_2 \in V$ definiert Äquivalenzrelation auf $K$-Vektorraum $V$. + +$v_1$ und $v_2$ sind Elemente einer Äquivalenzklasse gdw. sie sich um ein $u \in U$ unterscheiden. + +$V/U := \{[v] | v \in V\}$ mit $[v] := v+U := \{v+u|u \in U\}$ + +$dim(V/U) = dim(V) - dim(U)$ + +\section*{Eigenwerte} + +$A*v = \lambda * v$ wobei $\lambda \in Spec(A)$ und $v \in Eig_\lambda(A)$ + +\subsection*{Eigenräume} + +$Eig_\lambda(A) := \{v \in K^n : A*v = \lambda * v\} = Kern(A-\lambda I_n)$ + +\subsection*{Charakteristisches Polynom} + +$CP_A(x) = det(x*I_n - A)$ + +\subsection*{Minimalpolynom} + +$MP_\phi(x)$ ist Teiler kleinsten Grades von $CP_\phi(x)$ welcher jeden Linearfaktor beinhaltet und für den noch $MP_\phi(\phi) = 0$ gilt. + +\subsection*{Vielfachheit} + +$\mu_g(\phi, \lambda) := dim(Eig_\lambda(\phi))$ ist die geometrische Vielfachheit von $\lambda$. + +$\mu_a(\phi, \lambda)$ ist algebraische Vielfachheit von $\lambda$, also die Vielfachheit des Linearfaktors $(x-\lambda)$ von $CP_\phi$. + +\subsection*{Diagonalisierbarkeit} + +$\phi \in End(V)$ ist diagonalisierbar, wenn eines aus: + +\begin{enumerate}[label=(\alph*)] + \item $\exists$ Basis $B$ von $V$ aus Eigenvektoren von $\phi$ + \item $MP_\phi(X)$ zerfällt vollst. in Linearfaktoren + \item $CP_\phi(X)$ zerfällt vollst. in Linearfaktoren und $\forall \lambda \in Spec(\phi) : \mu_g(\phi,\lambda) = \mu_a(\phi, \lambda)$ +\end{enumerate} + +\subsubsection*{Diagonalisierungsverfahren} + +Eigenwerte und Eigenräume von $\phi$ bestimmen. Eigenwerte in $D_{CC}(\phi) = diag(\lambda_1, \hdots, \lambda_n)$ eintragen. $D_{CC}(\phi)$ ist Abbildungsmatrix von $\phi$ bzgl. Basis $C$ aus Eigenvektoren. $D_{BC}(Id)$ besteht aus Eigenvektoren in Spalten, $D_{CB}(Id) = D_{BC}(Id)^{-1}$. + +Insgesamt: $D_{CC}(\phi) = D_{BC}(Id)^{-1} D_{BB}(\phi) D_{BC}(Id)$. + +\section*{Haupträume} + +$H(\phi, \lambda) = Kern((\phi - \lambda * I_n)^e)$ mit $e := \mu_a(\phi, \lambda)$. + +Weiterhin gilt $dim(H(\phi, \lambda))=e$. + +\section*{Bilinearformen} + +\subsection*{Paarung, Bilinearform} + +$P : V \times W \rightarrow K$ ist Paarung von $V$ und $W$, wenn: + +\begin{enumerate}[label=(\alph*)] + \item $P(av_1 + v_2, w_1) = aP(v_1, w_1) + P(v_2, w_1)$ + \item $P(v_1, bw_1 + w_2) = bP(v_1, w_1) + P(v_1, w_2)$ +\end{enumerate} + +für $\forall a, b \in K$; $v_1, v_2 \in V$; $w_1, w_2 \in W$. Diese Eigenschaft wird als Bilinearität von $P$ bezeichnet. + +Falls $V=W$ heißt $P$ Bilinearform auf $V$. + +\subsubsection*{Ausartung} + +Paarung $P$ heißt nicht ausgeartet, wenn: + +\hspace*{2mm} +$\forall v \in V, v \neq 0 \exists w \in W : P(v, w) \neq 0$ + +$\land$ $\forall w \in W, w \neq 0 \exists v \in V : P(v, w) \neq 0$ + +\subsection*{Orthogonalbasis} + +Basis $B := \{b_1, ..., b_n\}$ ist Orthogonalbasis von $V$ bzgl. $P$, wenn: $\forall 1 \leq i \neq j \leq n : P(b_i, b_j) = 0$ + +\subsection*{Orthonormalbasis} + +Orthogonalbasis $B$ ist Orthonormalbasis von $V$ bzgl. $P$, wenn: $\forall 1 \leq i \leq n : P(b_i, b_i) = 1$ + +\section*{Jordansche Normalform $\tilde A = T^{-1} A T$} + +\subsection*{Darstellungsmatrix $\tilde A$ bzgl. Jordanbasis} + +\begin{enumerate}[leftmargin=4mm] + \item Eigenwerte aus char. Polynom bestimmen + \item Länge des Blocks zu Eigenwert ist $\mu_a(\lambda)$ + \item Anzahl Kästchen pro Block ist $\mu_g(\lambda)$ + \item Kleinstes $p$ mit $Kern((A-\lambda I)^p) = Kern((A-\lambda I)^{p+1})$ ist Länge des größten Kästchens zu $\lambda$ + \item Kästchen der Größe nach sortieren, Eigenwerte auf Hauptdiagonale + \item Anzahl der Jordankästchen mit Größe $s$ ergibt sich aus $2a_s - a_{s-1} - a_{s+1}$ mit $a_s = dim(Kern((A-\lambda I)^s))$ +\end{enumerate} + +\subsection*{Bestimmung Basiswechselmatrix $T$} + +Reichen die Eigenvektoren nicht aus, können Hauptvektoren hinzugezogen werden. Dazu wähle Hauptvektor $v_1$ $p$-ter Stufe wobei $p$ Länge des größten Kästchens zu $\lambda$ ist. $v_1$ kann direkt verwendet werden, weitere Vektoren ergeben sich als $v_{i+1} = (A-\lambda I)*v_i$. + +\section*{Skalarprodukte} + +\subsection*{Standardskalarprodukt auf $\R^n$} + +$\langle \cdot, \cdot \rangle : \R^n \times \R^n \rightarrow \R, \langle v, w \rangle := v^T * w$ + +\subsection*{Positive Definitheit} + +Eine symmetrische Bilinearform $\langle \cdot, \cdot \rangle : V \times V \rightarrow \R$ ist positiv definit, wenn: + +$\forall v \in V: v \neq 0 \Rightarrow \langle v, v \rangle > 0$ + +\subsection*{Skalarprodukt auf $V$} + +Ein Skalarprodukt auf $V$ ist symmetrische, positiv definite Bilinearform. Ein reeller Vektorraum mit Skalarprodukt ist euklidischer Vektorraum. + +\begin{description}[leftmargin=!,labelwidth=10mm] + \item[Norm] $||v|| := \sqrt{\langle v, v \rangle}$ + \item[Metrik] $d(v, w) := ||v - w||$ +\end{description} + +\subsection*{Komplexes Skalarprodukt} + +$\langle \cdot, \cdot \rangle : V \times V \rightarrow \mathbb{C}$ ist komplexes SKP, wenn: + +\begin{enumerate}[label=(\alph*)] + \item $\forall v_1, v_2, w \in V, a \in \mathbb{C} : \text{(linear)} \\ \hspace*{4mm} \langle av_1 + v_2, w \rangle = a \langle v_1, w \rangle + \langle v_2, w \rangle$ + \item $\forall v_1, v_2, w \in V, a \in \mathbb{C} : \text{(semilinear)} \\ \hspace*{4mm} \langle w, av_1 + v_2 \rangle = \overline a \langle w, v_1 \rangle + \langle w, v_2 \rangle$ + \item $\forall v, w \in V : \langle v, w \rangle = \overline{\langle w, v \rangle} \text{ (hermitesch)}$ + \item $\forall v \in V \setminus \{0\} : \langle v, v \rangle > 0$ +\end{enumerate} + +Wobei (a) und (b) Sesquilinearität, (c) Hermitizität und (d) Positivität. Ein komplexer Vektorraum mit komplexem SKP ist unitärer Vektorraum. + +\subsubsection*{Standardskalarprodukt auf $\mathbb{C}^n$} + +$\mathbb{C}^n \times \mathbb{C}^n \ni (v, w) \mapsto \langle v, w \rangle := v^T * \overline w = \sum_{i=1}^n v_i * \overline{w_i}$ + +\subsection*{Fundamentalmatrix} + +Seien $B$ und $C$ Basen von $V$ und $\langle \cdot, \cdot \rangle$ SKP. + +\vspace*{-3mm} +$$D_{BC}(\langle \cdot, \cdot \rangle) = \begin{pmatrix} + \langle b_1, c_1 \rangle & \hdots & \langle b_1, c_n \rangle \\ + \vdots & \ddots & \vdots \\ + \langle b_n, c_1 \rangle & \hdots & \langle b_n, c_n \rangle +\end{pmatrix}$$ + +\subsubsection*{Hurwitz-Kriterium} + +Eine symmetrische bzw. hermitesche Matrix $A$ ist positiv definit gdw. die Determinanten aller führenden Hauptminoren positiv sind. Führende Hauptminoren sind in der oberen linken Ecke beginnende quadr. Teilmatr. von $A$ inkl. $A$ selbst. + +\subsection*{Ungleichung von Cauchy-Schwarz} + +$\langle v, w \rangle ^2 \leq \langle v, v \rangle * \langle w, w \rangle$ (in $\R$) + +$|\langle v, w \rangle |^2 \leq \langle v, v \rangle * \langle w, w \rangle$ (in $\mathbb{C}$) + +\subsection*{Satz des Pythagoras} + +$||v+w||^2 = \langle v, v \rangle + 2\langle v, w \rangle + \langle w, w \rangle$ + +\subsubsection*{Orthogonalität} + +$v \perp w \Leftrightarrow ||v||^2 + ||w||^2 = ||v+w||^2$ + +\subsection*{Orthogonalisierung mit Gram-Schmidt} + +Sei $V$ euklidischer Vektorraum und $\{v_1, ..., v_k\} \subset V$ linear unabhängige Teilmenge mit $k$ Elementen. + +\vspace*{-4mm} +$$w_1 := v_1, w_l := v_l - \sum_{i=1}^{l-1} \frac{\langle v_l, w_i \rangle}{\langle w_i, w_i\rangle}*w_i \text{ (für } l = 2, ..., k)$$ + +Dann ist $S := \{w_1, ..., w_k\}$ Orthogonalsystem in $V$. + +$\tilde S := \{\frac{1}{||w_1||}*w_1, ..., \frac{1}{||w_k||}*w_k$ ist Orthonormalsystem. + +Im unitären Standardraum $\mathbb{C}^n$ ist Basis $B = \{b_1, ..., b_n\}$ ONB gd